Last year, pharmaceutical manufacturers significantly increased the amount of money they spent promoting new drugs, w...

Beraiah Genesis on January 11, 2020

Help

Can you please explain to me what the answer is and why. I don't understand it.

Reply
Create a free account to read and take part in forum discussions.

Already have an account? log in

Annie on January 16, 2020

Hi @beraiahgenesis,

This question is asking you to pick the answer choice which "most weakens" the argument. So, you're looking for an answer choice which messes up the logic the argument is using.

(A) is correct because it provides an alternative explanation for why the number of visits per sales representative fell last year. The conclusion of the argument says this was because physicians were less willing to receive their visits. But, this answer choice provides an different explanation, saying the number of visits per representative dropped because there were just more representatives.

(B) is incorrect because it doesn't challenge the logic of the argument. It is irrelevant to the number of visits representatives make.

(C) is incorrect because the question is about visits to doctors, not about the money spent on advertising to consumers.

(D) is incorrect because it doesn't deal with the central issue which is what changed from last year to this year so that sales representatives are making fewer visits. This answer choice could be about any year, so it doesn't affect the argument.

(E) is incorrect because it is like (D) in that it is not about the difference from last year to this year.